LSAT and Law School Admissions Forum

Get expert LSAT preparation and law school admissions advice from PowerScore Test Preparation.

 Administrator
PowerScore Staff
  • PowerScore Staff
  • Posts: 8916
  • Joined: Feb 02, 2011
|
#32468
Complete Question Explanation

Cannot Be True—PR, SN. The correct answer choice is (A)

This stimulus contains only a one-sentence principle. The principle is a conditional rule that uses the word “unless” to denote the necessary condition. We can use the Unless Equation to quickly and mechanically diagram this relationship. Whatever is modified by the word “unless” will be the necessary condition. We then negate the remainder of the relationship, which becomes the sufficient condition:


IMAB = should intentionally misrepresent another person’s beliefs
PAI = purpose is to act in the interest of the other person

..... ..... ..... ..... Sufficient ..... ..... Necessary

..... ..... ..... ..... IMAB ..... :arrow: ..... PAI


The question stem tells us this is a Cannot Be True—Principle question. Each answer choice will present a distinct factual scenario. The scenario in the correct answer choice will be directly inconsistent with the conditional rule diagrammed above, which provided that it is only permissible to intentionally misrepresent another person’s beliefs if your purpose is to act in the interest of the other person. Be careful—in order to violate this principle, the scenario must indicate that the sufficient condition has been met, and then violate the necessary condition. An answer choice that is inconsistent with the sufficient does not violate the principle, which is triggered only when the sufficient condition is satisfied. In other words, if the sufficient condition is not met, then there is no rule to apply.

Answer choice (A): This is the correct answer choice, because Ann’s actions violate the necessary condition of the principle in the stimulus. Ann’s purpose in misrepresenting Bruce’s belief about the Apollo missions was to make him look ridiculous, in violation of the rule’s necessary condition.

Answer choice (B): Although Claude misrepresented Thelma’s beliefs, he did so for purpose of keeping the person from bothering her, which we can assume was in Thelma’s interest.

Answer choice (C): Although we are told that John knew Maria would not want him to say what he did, we do not know whether he intentionally misrepresented her beliefs. So, the principle does not apply.

Answer choice (D): Here, Harvey misrepresented his own opinion, not Josephine’s, so the principle does not apply.

Answer choice (E): The scenario does not make clear that Wanda has misrepresented George’s beliefs. So, the principle does not apply.
 karina_fom
  • Posts: 7
  • Joined: Apr 24, 2016
|
#28238
Hello,

Could you please explain this question.

I chose B.

A is write answer but I don's understand why. I though "interest of that other person" is interest of person who is represented (in a case of A - Bruce), not the person who represent (Ann).
 David Boyle
PowerScore Staff
  • PowerScore Staff
  • Posts: 836
  • Joined: Jun 07, 2013
|
#28248
karina_fom wrote:Hello,

Could you please explain this question.

I chose B.

A is write answer but I don's understand why. I though "interest of that other person" is interest of person who is represented (in a case of A - Bruce), not the person who represent (Ann).

Hello karina_fom,

Answer B follows the principle in the stimulus, since Claude is trying to help Thelma by preventing some guy from bothering her. However, answer A violates the principle, since Ann is trying to hurt Bruce by making him look ridiculous. And that does follow what you said above, about "'interest of that other person' is interest of person who is represented (in a case of A - Bruce), not the person who represent (Ann)."

Hope this helps,
David
 karina_fom
  • Posts: 7
  • Joined: Apr 24, 2016
|
#28303
David,

Thank you for answering.
I realized that I misread the question - I thought they asked about answer choice that conforms this principle, not violates it.

K.
 MikeRov25
  • Posts: 18
  • Joined: Jan 14, 2016
|
#30882
David Boyle wrote:
karina_fom wrote:Hello,

Could you please explain this question.

I chose B.

A is write answer but I don's understand why. I though "interest of that other person" is interest of person who is represented (in a case of A - Bruce), not the person who represent (Ann).

Hello karina_fom,

Answer B follows the principle in the stimulus, since Claude is trying to help Thelma by preventing some guy from bothering her. However, answer A violates the principle, since Ann is trying to hurt Bruce by making him look ridiculous. And that does follow what you said above, about "'interest of that other person' is interest of person who is represented (in a case of A - Bruce), not the person who represent (Ann)."

Hope this helps,
David
Hi David, I did the same thing as karina and misread the question when I took the test. I understand how A is the correct answer, but my question is why wouldn't E work? Is it because A is stronger in that "she wanted to make him look ridiculous" instead of "she wanted people to know that George knew little about geography".
User avatar
 Jonathan Evans
PowerScore Staff
  • PowerScore Staff
  • Posts: 726
  • Joined: Jun 09, 2016
|
#30904
Hi, MikeRov,

Answer Choice (E) is actually incorrect because we don't know whether Wanda is misrepresenting George's beliefs. For all we know George does think Egypt is in Asia. Wanda might be acting like a jerk, but George could be ignorant too! Hence, this answer doesn't necessarily violate the principle.
 MikeRov25
  • Posts: 18
  • Joined: Jan 14, 2016
|
#31048
Ah I gotcha! Thanks!
 Pragmatism
  • Posts: 68
  • Joined: Jan 11, 2018
|
#43329
So, I chose the completely wrong answer choice, D. Nevertheless, my question was regarding the nuance between answer choice A and E. I wanted to know, if my reasoning would likely veer me in the right direction moving forward.

So, both answer choices mention, "told someone," the person they were talking about thought something, and that the did this act not to benefit the person they were speaking ill about. The difference between the right and wrong answer here is the phrase, "but she told people that George thought it was in Asia," whereas, in answer choice A, "even though she knew he did not think this," am I right? Because, in answer choice E, Wanda never mentions what George think's or believes, whereas that contrast is made in answer choice A.
 Adam Tyson
PowerScore Staff
  • PowerScore Staff
  • Posts: 5153
  • Joined: Apr 14, 2011
|
#43643
You are pretty much there, pragmatism! The issue missing in answer E but present in answer A is that of misrepresentation. In A, we KNOW that Ann is misrepresenting Bruce's beliefs, but in answer E we don't know what George believed. If he believed that Egypt was in Asia, then Wanda didn't misrepresent his belief, and the principle isn't violated because it never even comes into play! That's all it takes to make A the better choice.

Well done, my pragmatic friend!
 Pragmatism
  • Posts: 68
  • Joined: Jan 11, 2018
|
#43661
Thank you so much Adam.

Get the most out of your LSAT Prep Plus subscription.

Analyze and track your performance with our Testing and Analytics Package.